Tải bản đầy đủ (.pdf) (23 trang)

nesbit

Bạn đang xem bản rút gọn của tài liệu. Xem và tải ngay bản đầy đủ của tài liệu tại đây (180.87 KB, 23 trang )

<span class='text_page_counter'>(1)</span><div class='page_container' data-page=1></div>
<span class='text_page_counter'>(2)</span><div class='page_container' data-page=2>

Tháng 3/1903, trên tạp chíEducational T imes, A.M.Nesbitt đã đ-a ra bài tốn:
Cho a, b, c là các số thực d-ơng. Chứng minh rằng:


a
b+c +


b
c+a +


c
a+b


3
2


Đẳng thức xảy ra khi và chỉ khi a=b =c.


Bất đẳng thức trên đ-ợc gọi là bất đẳng thức Nesbitt. Đây là bất đẳng thức đẹp
và đã thu hút đ-ợc sự chú ý của nhiều ng-ời. Trong bài viết này, tơi xin nói về những
ứng dụng, mở rộng và một số vấn đề liên quan đến nó.


1 Bất đẳng thức Nesbitt và ứng dụng



Nh- ta đã biết, bất đẳng thứcNesbitt là một bất đẳng thức cơ bản, có nhiều ứng dụng
quan trọng trong giải tốn. Sau đây, tơi xin giới thiệu một số ví dụ để làm rõ hơn về
điều đó.


VÝ dơ 1.1. Choa, b, c >0 tho¶ m·nabc= 1. Chøng minh r»ng:


1



a2<sub>(</sub><sub>b</sub><sub>+</sub><sub>c</sub><sub>)</sub> +


1


b2<sub>(</sub><sub>c</sub><sub>+</sub><sub>a</sub><sub>)</sub> +


1


c2<sub>(</sub><sub>a</sub><sub>+</sub><sub>b</sub><sub>)</sub> ≥


3
2


Lêi gi¶i. Ta có:


X 1


a2<sub>(</sub><sub>b</sub><sub>+</sub><sub>c</sub><sub>)</sub> =


X abc
a2<sub>(</sub><sub>b</sub><sub>+</sub><sub>c</sub><sub>)</sub> =


X bc
ab+ca


3
2


Đẳng thức xảy ra khi vµ chØ khi a=b =c= 1.



VÝ dơ 1.2. Choa, b, c >0 tho¶ m·nabc= 1. Chøng minh r»ng:
a


(b+c)2 +


b


(c+a)2 +


c


(a+b)2 ≥


9
4 (a+b+c)


Lời giải. Ta viết lại bất đẳng thức:


(a+b+c)



a


(b+c)2 +


b


(c+a)2 +


c



(a+b)2




≥ 9
4


Theo bất đẳng thức Cauchy<sub>−</sub>Schwarz có:


(a+b+c)



a


(b+c)2 +


b


(c+a)2 +


c


(a+b)2








a
b+c+


b
c+a +


c
a+b


2


≥ 9<sub>4</sub>


</div>
<span class='text_page_counter'>(3)</span><div class='page_container' data-page=3>

VÝ dô 1.3. Choa, b, c >0 tho¶ m·nabc= 1. Chøng minh r»ng:


1


a(b+ 1) +
1


b(c+ 1) +
1


c(a+ 1)
3
2


Lời giải. Đặt a=x/y, b=y/z, c=z/x, ta có:
X 1



a(b+ 1) =


X yz
xy+zx


3
2


Đẳng thức xảy ra khi và chØ khi a=b =c= 1.


VÝ dơ 1.4 (§Ị thi Olympic 30 - 4). Choa, b > 0vµ x, y, z là các số d-ơng tuỳ ý.
Tìm giá trị nhỏ nhất cđa:


x2


(ay+bz)(az+by)+


y2


(az+bx)(ax+bz) +


z2


(ax+by)(ay+bx)


Lời giải. Theo bất đẳng thức AM <sub>−</sub>GM có:


(ay+bz)(az+by)<sub>≤</sub> (ay+bz+az+by)


2



4 =


(a+b)2<sub>(</sub><sub>y</sub><sub>+</sub><sub>z</sub><sub>)2</sub>


4 ≤


(a+b)2<sub>(</sub><sub>y</sub>2<sub>+</sub><sub>z</sub>2<sub>)</sub>


2


Suy ra,


x2


(ay+bz)(az+by) ≥


2x2


(a+b)2<sub>(</sub><sub>y</sub>2<sub>+</sub><sub>z</sub>2<sub>)</sub>


T-¬ng tù, ta cã:


y2


(az+bx)(ax+bz) ≥


2y2


(a+b)2<sub>(</sub><sub>z</sub>2<sub>+</sub><sub>x</sub>2<sub>)</sub>



z2


(ax+by)(ay+bx) ≥


2z2


(a+b)2<sub>(</sub><sub>x</sub>2<sub>+</sub><sub>y</sub>2<sub>)</sub>


Do ú,


X x2


(ay+bz)(az+by)
2
(a+b)2



x2
y2<sub>+</sub><sub>z</sub>2 +


y2
z2<sub>+</sub><sub>x</sub>2 +


z2
x2<sub>+</sub><sub>y</sub>2




<sub>(</sub><sub>a</sub><sub>+</sub>3<sub>b</sub><sub>)</sub>2



Đẳng thức xảy ra khi vµ chØ khi x=y=z.


Bất đẳng thức Nesbittkhơng chỉ ứng dụng trong các bài bất đẳng thức Đại số mà còn
là một cơng cụ quan trọng trong các bài tốn bất đẳng thức Hình học.


VÝ dơ 1.5. Chøng minh r»ng:
ma


lb+hc


+ mb


lc+ha


+ mc


</div>
<span class='text_page_counter'>(4)</span><div class='page_container' data-page=4>

H-íng dÉn. Tr-íc hÕt ta chøng minh: ha≤la ≤ma.


Từ đó ta có:


X m<sub>a</sub>
lb+hc ≥


X m<sub>a</sub>
mb+mc ≥


3
2



Đẳng thức xảy ra khi và chỉ khi <sub>4</sub>ABC đều.


VÝ dô 1.6. Cho tam giác ABC có 3 đ-ờng phân giácAA1, BB1, CC1. Gọi khoảng cách


từ A1 dến AB, B1 dến BC, C1 dếnCA lần l-ợt là a1, b1, c1. Chứng minh rằng:


a1


ha
+ b1


hb
+ c1


hc
3
2


Lời giải. Gọi H là chân đ-ờng vuông góc hạ từ A xuống BC và K là chân đ-ờng
vuông góc hạ từ A1 xuốngAB.


Ta có:


SABA1 =


1


2ha.BA1=
1
2a1.AB


a1


ha


= BA1


AB =
CA1


CA =


BA1+CA1


AB+CA =
a
b+c
T-¬ng tù, ta cã:


b1


hb


= b


c+a;
c1


hc


= c



a+b
Do đó,


a1


ha
+ b1


hb
+ c1


hc


= a


b+c+
b
c+a +


c
a+b ≥


3
2


</div>
<span class='text_page_counter'>(5)</span><div class='page_container' data-page=5>

VÝ dô 1.7. Cho tam giácABC nội tiếp(O). Đ-ờng phân giác trong gócA cắtBC tại
A1, cắt (O) tại A2. Các điểm B1, B2;C1, C2 đ-ợc đinh nghĩa t-ơng tự A1, A2. Chứng


minh rằng:



A1A2


BA2+CA2


+ B1B2


AB2+CB2


+ C1C2


AC2+BC2 ≥


3
4


Lời giải. Vì đ-ờng phân giác trong góc A cắt (O) tại A2 nên BA2 =CA2. Do đó:


A1A2


BA2+CA2


= A1A2
2CA2


Dễ dàng chứng minh đ-ợc <sub>4</sub>CA1A2 4ACA2. Suy ra: A<sub>CA</sub>1A<sub>2</sub>2 = CA<sub>AA</sub>2<sub>2</sub>.


Tứ giác ABA2C nội tiếp, theo định lí P tolemecú:


BC.AA2=AB.CA2+AC.BA2



BC.AA2=CA2(AB+CA)


CA2


AA2


= BC


AB+CA =
a
b+c
Tóm lại,


A1A2


BA2+CA2


= a


2(b+c)


T-ơng tự, ta có:


B1B2


AB2+CB2


= b



2(c+a);


C1C2


AC2+BC2


= c


2(a+b)


Do đó,
A1A2


BA2+CA2


+ B1B2


AB2 +CB2


+ C1C2


AC2+BC2


= 1
2



a
b+c+



b
c+a +


c
a+b




≥ 3
4


</div>
<span class='text_page_counter'>(6)</span><div class='page_container' data-page=6>

Nhận xét. Trong cách chứng minh trên ta mới sử dụng đẳng thức P toleme. Nếu sử
dụng bất đẳng thức P toleme thì ta có bài tốn tổng qt hơn:


VÝ dơ 1.8. Cho lơc gi¸c ABCDEF cã AB = BC, CD = DE, EF = F A. Chøng
minh r»ng:


BC
BE +


DE
AD +


F A
CF ≥


3
2


VÝ dơ 1.9(§Ị thi Olympic 30 - 4, 2003). Cho tam giácABC nội tiếp (O). Các đ-ờng


trung tuyến AA1, BB1, CC1 lần l-ợt cắt(O) tại A2, B2, C2. Chøng minh r»ng:


AA1


AA2


+ BB1


BB2


+CC1


CC2 ≤


9
4


H-íng dÉn. Ta dƠ dµng cã:


AA1.A1A2 =BA1.CA1=a2/4


AA2<sub>1</sub> = 2b


2<sub>+ 2</sub><sub>c</sub>2<sub>−</sub><sub>a</sub>2


4


Suy ra,


AA1



AA2


= 2b


2<sub>+ 2</sub><sub>c</sub>2<sub>−</sub><sub>a</sub>2


2b2<sub>+ 2</sub><sub>c</sub>2 = 1−


a2


2(b2<sub>+</sub><sub>c</sub>2<sub>)</sub>


T-¬ng tù, ta cã:


BB1


BB2


= 1<sub>−</sub> b


2


2(c2<sub>+</sub><sub>a</sub>2<sub>)</sub>;


CC1


CC2


= 1<sub>−</sub> c



2


2(a2<sub>+</sub><sub>b</sub>2<sub>)</sub>


VËy,


AA1


AA2


+BB1


BB2


+ CC1


CC2


= 3<sub>−</sub> 1
2




X a2
b2<sub>+</sub><sub>c</sub>2




≤ 9<sub>4</sub>



</div>
<span class='text_page_counter'>(7)</span><div class='page_container' data-page=7>

2 Bất đẳng thức Nesbitt và mở rộng



Bất đẳng thức Nesbitt có rất nhiều ứng dụng quan trọng trong giải tốn nên việc mở
rộng nó là một công việc cần thiết. Trong mục này, tôi sẽ đ-a ra mở rộng bất đẳng thức
Nesbitttheo hai h-ớng lànhững mở rộng trực tiếpvà những mở rộng có thêm tham số.


2.1 Nh÷ng më réng trùc tiÕp



Đầu tiên, chúng ta sẽ nghĩ ngay đến việc kéo dài bất đẳng thức Nesbitt.


Më réng 1. Cho a1, a2, ..., an>0; n≥2. Chøng minh r»ng:


X a<sub>1</sub>


a2+a3+...+an ≥


n


n<sub>−</sub>1 (1)


Lời giải. Giả sử a1 ≥a2 ≥...≥an. Khi đó:
1


a2+a3+...+an ≥


1


a1+a3+...+an ≥



...<sub>≥</sub> 1


a1+a2+...+an−1


Theo bất đẳng thức Chebyshev cú:
X a<sub>1</sub>


a2+a3+...+an
1


n(a1+a2+...+an)


X 1


a1+a2+...+an1




<sub>n</sub>1(a1+a2+...+an)




n2


(a1+a2+...+an)(n1)




= n



n<sub></sub>1


Đẳng thức xảy ra khi và chỉ khi a1 =a2 =...=an.


Nếu gắn thêm số mũ vµo Më réng 1. ta cã:


Më réng 2. Cho a1, a2, ..., an>0; n≥2 vµ k≥ (n−1)/n. Chøng minh r»ng:


X a<sub>1</sub>


a2+a3+...+an


k


≥ <sub>(</sub><sub>n</sub> n


−1)k (2)


Ta xét tiếp đến một mở rộng nữa về chiều dài.


Më réng 3. Víi mäi xi ≥0, xi+xi+1 >0, xn+i =xi (i= 1,2, ..., n) th×:
n


X


i=1


xi



xi+1+xi+2 ≥


n


</div>
<span class='text_page_counter'>(8)</span><div class='page_container' data-page=8>

Chú ý. Bất đẳng thức trên là bất đẳng thứcShapiro đ-ợc nhà Toán học Shapiro đ-a
ra trên tạp chí American Mathematic Monthly năm 1954. Bất đẳng thức Shapironhìn
rất đơn giản nh-ng việc chứng minh lại vơ cùng khó vì nó khơng đúng với mọi số tự
nhiên n. Tuy nhiên cuối cùng thì nhà Tốn học T roeschđã chứng minh đ-ợc bất đẳng
thức Shapiro với kết quả quan trọng sau:


Bất đẳng thức Shapiro đúng với mọi n chẵn <sub>≤</sub> 12 và n lẻ <sub>≤</sub> 23. Với mọi giá trị khác
của n thì bất đẳng thức sai.


Mở rộng bất đẳng thức Nesbittth-ờng gặp là gắn với số mũ. Ta xét một vài mở rộng:


Më réng 4. Cho a, b, c >0 và n<sub></sub>1. Chứng minh rằng:
an


b+c+
bn


c+a +
cn


a+b


an1<sub>+</sub><sub>b</sub>n1<sub>+</sub><sub>c</sub>n1


2



3
2




a+b+c


3


n1


(4)


Hệ quả 1. Với a, b, c >0 thoả mÃn abc= 1và n<sub></sub> 1thì:
an


b+c +
bn


c+a +
cn


a+b


3
2


Ta có thể gắn thêm hệ sè vµo Më réng 4. nh- sau:


Më réng 5. Cho a, b, c >0 vµ n<sub>≥</sub>1. Chøng minh r»ng:


an


pb+qc+
bn


pc+qa+
cn


pa+qb ≥


an−1 <sub>+</sub><sub>b</sub>n−1<sub>+</sub><sub>c</sub>n−1


p+q ≥


3


p+q


a+b+c


3


n−1
(5)
Chứng minh. Theo bất đẳng thức Cauchy<sub>−</sub>Schwarz có:


an


pb+qc +


bn


pc+qa+
cn


pa+qb


= a


2n−2


pban−2<sub>+</sub><sub>qca</sub>n−2 +


b2n−2


pcbn−2 <sub>+</sub><sub>qab</sub>n−2 +


c2n−2


pacn−2<sub>+</sub><sub>qbc</sub>n−2


≥ (a


n−1<sub>+</sub><sub>b</sub>n−1<sub>+</sub><sub>c</sub>n−1<sub>)</sub>2


p(ban−2<sub>+</sub><sub>cb</sub>n−2<sub>+</sub><sub>ac</sub>n−2<sub>) +</sub><sub>q</sub><sub>(</sub><sub>ca</sub>n−2<sub>+</sub><sub>ab</sub>n−2<sub>+</sub><sub>bc</sub>n−2<sub>)</sub>


Theo bất đẳng thức hoán vị có:


an−1<sub>+</sub><sub>b</sub>n−1<sub>+</sub><sub>c</sub>n−1



≥ban−2<sub>+</sub><sub>cb</sub>n−2 <sub>+</sub><sub>ac</sub>n−2


an−1<sub>+</sub><sub>b</sub>n−1<sub>+</sub><sub>c</sub>n−1 <sub>≥</sub><sub>ca</sub>n−2<sub>+</sub><sub>ab</sub>n−2<sub>+</sub><sub>bc</sub>n−2


Do đó,


an


pb+qc+
bn


pc+qa+
cn


pa+qb ≥


an−1<sub>+</sub><sub>b</sub>n−1<sub>+</sub><sub>c</sub>n−1


</div>
<span class='text_page_counter'>(9)</span><div class='page_container' data-page=9>

ThËt dƠ dàng chứng minh đ-ợc:


an1<sub>+</sub><sub>b</sub>n1<sub>+</sub><sub>c</sub>n1


p+q


3


p+q



a+b+c


3


n1


Từ đây ta suy ra điều phải chứng minh.
Đẳng thức xảy ra khi và chỉ khi a=b =c.


Nếu nâng số mũ mẫu số của Mở rộng 4. thì ta có bài tốn t-ơng đối tổng qt sau:


Më réng 6. Cho a, b, c >0. Chøng minh r»ng:
an


(b+c)m +


bn
(c+a)m +


cn
(a+b)m ≥


an−m<sub>+</sub><sub>b</sub>n−m <sub>+</sub><sub>c</sub>n−m


2m (6)


Chøng minh. Ta cã: (b+c)m<sub>≤</sub> <sub>2</sub>m1<sub>(</sub><sub>b</sub>m<sub>+</sub><sub>c</sub>m<sub>)</sub><sub>, suy ra:</sub>


X



cyc


an
(b+c)m


X


cyc


an
2m1<sub>(</sub><sub>b</sub>m<sub>+</sub><sub>c</sub>m<sub>)</sub>


Bây giờ, ta chỉ cần chứng minh:
X


cyc


an


2m1<sub>(</sub><sub>b</sub>m<sub>+</sub><sub>c</sub>m<sub>)</sub> ≥


an−m<sub>+</sub><sub>b</sub>n−m<sub>+</sub><sub>c</sub>n−m
2m


ThËt vËy, ta cã:
X


cyc


an



2m−1<sub>(</sub><sub>b</sub>m<sub>+</sub><sub>c</sub>m<sub>)</sub>−


an−m<sub>+</sub><sub>b</sub>n−m <sub>+</sub><sub>c</sub>n−m
2m


= X


cyc


an−m



am


bm<sub>+</sub><sub>c</sub>m −
1
2




= X


cyc


an−m


bm<sub>+</sub><sub>c</sub>m(2a
m



−bm<sub>−</sub>cm)


= X


cyc


(am<sub>−</sub>bm)




an−m


bn−m<sub>+</sub><sub>c</sub>n−m −


bn−m


cn−m<sub>+</sub><sub>a</sub>n−m




= X


cyc


am<sub>−</sub><sub>b</sub>m


(bn−m<sub>+</sub><sub>c</sub>n−m<sub>)(</sub><sub>c</sub>n−m <sub>+</sub><sub>a</sub>n−m<sub>)</sub> c


n−m<sub>(</sub><sub>a</sub>n−m



−bn−m<sub>) +</sub><sub>a</sub>2(n−m)


−b2(n−m)


≥ 0


</div>
<span class='text_page_counter'>(10)</span><div class='page_container' data-page=10>

Më réng 7. Cho a, b, c >0 vµ n lµ h»ng sè cho tr-íc. Chøng minh r»ng:


a
b+c


n


+



b
c+a


n


+



c
a+b


n







3
2n; 2




(7)
H-ớng dẫn. Bài tốn trên dễ dàng chứng minh đ-ợc với tr-ờng hợp n <sub>≤</sub> 0 và n <sub>≥</sub> 1.
Còn với tr-ờng hợp 0< n <1, ta sẽ chứng minh đ-ợc bằng ph-ơng pháp dồn biến.
Hằng số tốt nhất cho bất đẳng thức là ln 3


ln 21.


Chú ý. Đây là kết quả ở [1].


Hệ quả 2. Víi a, b, c >0 vµ n <sub>∈</sub>N, n<sub>≥</sub>2 th×:


n


r
a
b+c +


n


r
b


c+a +


n


r
c
a+b >


n
n<sub>−</sub>1


n




n<sub>−</sub>1


Bất đẳng thức Nesbitt có một dạng mở rộng nữa khi thêm hệ số ở tử số.


Më réng 8. Cho m, n, p;x, y, z >0. T×m giá trị nhỏ nhất của:
ma


b+c +
nb
c+a +


pc


a+b (8)



Chứng minh. Đặt


x=b+c, y=c+a, z =a+b


⇒ a= y+z−x


2 , b=


z+x<sub>−</sub>y


2 , c =


x+y<sub>−</sub>z


2


Suy ra,


ma
b+c+


nb
c+a +


pc
a+b


= m(y+z−x)


2x +



n(z+x<sub>−</sub>y)


2y +


p(z+y<sub>−</sub>z)
2z


= 1
2



my


x +
nx


y


+my


x +
my


x


+my



x +
my


x


−(m+n+p)




≥ √mn+√mp+√pn<sub>−</sub> m+n+p


2


VËy,


ma
b+c+


nb
c+a +


pc
a+b ≥




mn+√mp+√pn<sub>−</sub>m+n+p


2



</div>
<span class='text_page_counter'>(11)</span><div class='page_container' data-page=11>

2.2 Nh÷ng më réng cã thêm tham số



Bài toán 2.2.1. Cho a, b, c;x, y, z >0. Chøng minh r»ng:
a


b+cx


2<sub>+</sub> b


c+ay


2<sub>+</sub> c


a+bz


2 <sub>≥</sub><sub>xy</sub><sub>+</sub><sub>yz</sub><sub>+</sub><sub>zx</sub><sub>−</sub> 1


2(x


2<sub>+</sub><sub>y</sub>2<sub>+</sub><sub>z</sub>2<sub>)</sub>


Chứng minh. Cộng mỗi vế của bất đẳng thức với x2<sub>+</sub><sub>y</sub>2<sub>+</sub><sub>z</sub>2<sub>, ta có:</sub>


a
b+cx


2<sub>+</sub><sub>x</sub>2<sub>+</sub> b


c+ay



2<sub>+</sub><sub>y</sub>2<sub>+</sub> c


a+bz


2<sub>+</sub><sub>z</sub>2


≥xy+yz+zx+ 1
2(x


2<sub>+</sub><sub>y</sub>2<sub>+</sub><sub>z</sub>2<sub>)</sub>


Ta viết lại bất đẳng thức:


(a+b+c)



x2


b+c+
y2


c+a+
z2


a+b


≥xy+yz+zx+1
2(x



2<sub>+</sub><sub>y</sub>2<sub>+</sub><sub>z</sub>2<sub>)</sub>


Theo bất ng thc Cauchy<sub></sub>Schwarz cú:
x2


b+c+
y2


c+a +
z2


a+b


(x+y+z)2


2(a+b+c)


Từ đây ta suy ra điều phải chứng minh.
Đẳng thức xảy ra khi và chỉ khi b+c


x =
c+a


y =
a+b


z .


Từ bài toán trên, ta có thể suy ra bài toán sau:



Bài toán 2.2.2. Cho u, v, w >0 và a, b, c là ba cạnh cđa tam gi¸c cã diƯn tÝchS.
Chøng minh r»ng:


u
v +wa


2<sub>+</sub> v


w+ub


2<sub>+</sub> w


u+vc


2


23S
H-ớng dẫn. Ta chỉ cần chứng minh:


ab+bc+ca<sub></sub> 1


2(a


2<sub>+</sub><sub>b</sub>2<sub>+</sub><sub>c</sub>2<sub>)</sub>


23S
Đặt x=b+c<sub></sub>a, y=c+a<sub></sub>b, z =a+b<sub></sub>c, ta có:


ab+bc+ca<sub></sub> 1



2(a


2 <sub>+</sub><sub>b</sub>2<sub>+</sub><sub>c</sub>2<sub>) =</sub><sub>xy</sub><sub>+</sub><sub>yz</sub><sub>+</sub><sub>zx</sub>


S = 1
4


p


xyz(x+y+z)


Từ đây ta suy ra điều phải chứng minh.


</div>
<span class='text_page_counter'>(12)</span><div class='page_container' data-page=12>

Bài to¸n 2.2.3. Cho a, b, c;x, y, z >0. Chøng minh r»ng:
a


b+c(y+z) +
b


c+a(z+x) +
c


a+b(x+y)≥


X p


(x+y)(x+z)<sub>−</sub>(x+y+z)


Chứng minh. Theo bất đẳng thức Cauchy<sub>−</sub>Schwarz có:


a


b+c(y+z) +
b


c+a(z+x) +
c


a+b(x+y)


= (a+b+c)



y+z


b+c +
z+x
c+a +


x+y
a+b




−2(x+y+z)
≥ 1<sub>2</sub> √y+z+√z+x+√x+y2


−2(x+y+z)
= X p(x+y)(x+z)<sub>−</sub>(x+y+z)



Bất đẳng thức đã đ-ợc chứng minh xong.


NhËn xÐt. Ta có thể chứng minh đ-ợc:
X p


(x+y)(x+z)<sub></sub>(x+y+z)<sub></sub>p3(xy+yz+zx)<sub></sub>3




xy+yz+zx
x+y+z




T ú, ta cú hai bi toỏn h qu sau:


Bài toán 2.2.4. Cho a, b, c;x, y, z >0. Chứng minh rằng:
a


b+c(y+z) +
b


c+a(z+x) +
c


a+b(x+y)3


xy+yz+zx
x+y+z





Bài toán 2.2.5. Cho a, b, c;x, y, z >0. Chøng minh r»ng:
a


b+c(y+z) +
b


c+a(z+x) +
c


a+b(x+y)≥
p


3(xy+yz+zx)


Bình luận. Trong phần trên, tơi đã đ-a ra một vài bài tốn có gắn thêm hằng số vào
bất đẳng thức Nesbitt. Những bài tốn đó th-ờng khơng đ-ợc coi là mở rộng của bất
đẳng thức Nesbitt. Tuy nhiên theo quan điểm của tơi thì nó vẫn là một dạng mở rộng
bởi vì khi lấy một giá trị đặc biệt thay cho những hằng số đó thì ta sẽ thu đ-ợc bất
đẳng thức Nesbitt. Ví dụ, ở Bài tốn 2.2.3 ta chọnx=y=z thì nó sẽ trở thành:


a
b+c +


b
c+a +


c


a+b ≥


</div>
<span class='text_page_counter'>(13)</span><div class='page_container' data-page=13>

3 Tản mạn bất đẳng thức Nesbitt



3.1 So sỏnh cỏc bt ng thc dng Nesbitt



Bài toán 3.1.1. Cho a, b, c >0 vµ m<sub>≥</sub>n <sub>≥</sub>0. Chøng minh rằng:
am


bm<sub>+</sub><sub>c</sub>m +


bm


cm<sub>+</sub><sub>a</sub>m +


cm


am<sub>+</sub><sub>b</sub>m


an


bn<sub>+</sub><sub>c</sub>n +


bn


cn<sub>+</sub><sub>a</sub>n +


cn


an<sub>+</sub><sub>b</sub>n



Lời giải 1. Đặt


f(t) = a
t


bt<sub>+</sub><sub>c</sub>t +


bt


ct<sub>+</sub><sub>a</sub>t +


ct


at<sub>+</sub><sub>b</sub>t


Bây giờ, ta chỉ việc chứng minh cho hàm số f(t) đơn điệu tăng theo t<sub>≥</sub>0.
Dễ dàng ta cú:


f0<sub>(</sub><sub>t</sub><sub>) =</sub>X


sym


atbt(at<sub></sub>bt)(lna<sub></sub>lnb) 2c


t<sub>+</sub><sub>a</sub>t<sub>+</sub><sub>b</sub>t


(bt<sub>+</sub><sub>c</sub>t<sub>)</sub>2<sub>(</sub><sub>a</sub>t<sub>+</sub><sub>c</sub>t<sub>)</sub>2 0


Từ đây ta suy ra điều phải chứng minh.


Đẳng thức xảy ra khi và chỉ khi a=b =c.
Lêi gi¶i 2. Ta cã:


X am
bm<sub>+</sub><sub>c</sub>m ≥


X an
bn<sub>+</sub><sub>c</sub>n


⇔ X a


m


bm<sub>+</sub><sub>c</sub>m −


an


bn<sub>+</sub><sub>c</sub>n ≥0


⇔ Xa


m<sub>(</sub><sub>b</sub>n<sub>+</sub><sub>c</sub>n<sub>)</sub><sub>−</sub><sub>a</sub>n<sub>(</sub><sub>b</sub>m<sub>+</sub><sub>c</sub>m<sub>)</sub>
(bm<sub>+</sub><sub>c</sub>m<sub>)(</sub><sub>b</sub>n<sub>+</sub><sub>c</sub>n<sub>)</sub> ≥0


⇔ Xa


n<sub>b</sub>n<sub>(</sub><sub>a</sub>m−n<sub>−</sub><sub>b</sub>m−n<sub>) +</sub><sub>a</sub>n<sub>c</sub>n<sub>(</sub><sub>a</sub>m−n<sub>−</sub><sub>c</sub>m−n<sub>)</sub>
(bm<sub>+</sub><sub>c</sub>m<sub>)(</sub><sub>b</sub>n<sub>+</sub><sub>c</sub>n<sub>)</sub> ≥0
⇔ Xanbn(am−n



−bm−n<sub>)</sub>




1


(bm<sub>+</sub><sub>c</sub>m<sub>)(</sub><sub>b</sub>n<sub>+</sub><sub>c</sub>n<sub>)</sub> −


1


(am<sub>+</sub><sub>c</sub>m<sub>)(</sub><sub>a</sub>n<sub>+</sub><sub>c</sub>n<sub>)</sub>




≥0 (*)
Dễ thấy (*) luôn đúng. Từ đây ta suy ra điều phải chứng minh. Đẳng thức xảy ra khi
và chỉ khi a=b=c.


Bài toán 3.1.2. Cho a1, a2, ..., an>0. Chứng minh rằng:


a2
1


a2


2+...+a2n


+...+ a


2



n


a2


1+...+a2n1


a1


a2+...+an


+...+ an


</div>
<span class='text_page_counter'>(14)</span><div class='page_container' data-page=14>

Lời giải. Đặt


f(t) =
n


X


i=1


at


1


at


2+at3+...+atn



S =at<sub>1</sub>+at<sub>2</sub>+...+at<sub>n</sub>


Ta sẽ chứng minh cho hàm số f(t)đơn điệu tăng theo t <sub>≥</sub>0. Ta có:
f0<sub>(</sub><sub>t</sub><sub>) =</sub>


n


X


1


at


1lna1(at2+a3t +...+atn)−at1(at2lna2+at3lna3+...+atnlnan)
(S<sub>−</sub>at


1)2


f0<sub>(</sub><sub>t</sub><sub>) =</sub>


n


X


1


at


1at2(lna1−lna2) +at1at3(lna1−lna3) +...+at1atn(lna1−lnan)



(S<sub>−</sub>at


1)2


Ta viÕt l¹i f0<sub>(</sub><sub>t</sub><sub>)</sub> <sub>d-íi d¹ng:</sub>


f0<sub>(</sub><sub>t</sub><sub>) =</sub>


n


X


i,j=1


at


iatj(lnai−lnaj)
(S<sub>−</sub>at


i)2


=
n


X


i,j=1


at<sub>i</sub>at<sub>j</sub>(lnai−lnaj)





1
(S<sub>−</sub>at


i)2 −
1
(S<sub>−</sub>at


j)2




=
n


X


i,j=1


at<sub>i</sub>at<sub>j</sub>(lnai−lnaj)(ati−atj)


2S<sub>−</sub>at
i −atj
(S<sub>−</sub>at


i)2 + (Satj)2
0


Từ đây ta suy ra điều phải chứng minh.



Đẳng thức xảy ra khi và chỉ khi mỗi số ai hoặc b»ng nhau hc b»ng 0.


Bình luận. Đến đây chắc hẳn các bạn đang đặt ra câu hỏi liệu rằng ta có thể kết hợp
hai bất đẳng thức trên để có đ-ợc một bất đẳng thức tổng quát hơn không? Đây là một
ý nghĩ hết sức tự nhiên. Và tôi hy vọng các bạn sẽ suy nghĩ để tìm ra câu trả lời. Chúc
các bạn thành công!


3.2 Liệu đã chặt?



Đã bao giờ bạn đặt ra câu hỏi con số 3/2 đã chặt với bất đẳng thứcNesbittch-a? Câu
trả lời là 3/2 ch-a phải là con số thực sự chặt! Tôi xin lấy một vài ví dụ.


Bài tốn 3.2.1. Cho a, b, c khơng âm. Tìm hằng số k tốt nhất để bất đẳng thức sau
đúng:


a
b+c +


b
c+a +


c
a+b ≥


3
2+


k.max<sub>{</sub>(a<sub>−</sub>b)2<sub>,</sub><sub>(</sub><sub>b</sub><sub>−</sub><sub>c</sub><sub>)</sub>2<sub>,</sub><sub>(</sub><sub>c</sub><sub>−</sub><sub>a</sub><sub>)</sub>2<sub>}</sub>



</div>
<span class='text_page_counter'>(15)</span><div class='page_container' data-page=15>

Lời giải. Khơng mất tính tổng qt, giả sử a <sub>≥</sub>b <sub>≥</sub>c, bất đẳng thức trở thành:
a


b+c+
b
c+a +


c
a+b ≥


3
2+


k.(a<sub>−</sub>c)2


ab+bc+ca
Chon a = 4/3, b = 1, c = 0, ta cã: k <sub>≤</sub>7/16.


Ta sẽ chứng minh cho đây là giá trị cần tìm, nghÜa lµ:
a


b+c+
b
c+a +


c
a+b ≥


3
2 +



7(a<sub>−</sub>c)2


16(ab+bc+ca)


⇔ X


cyc


a(ab+ac+bc)


b+c ≥


3(ab+bc+ca)


2 +


7(a<sub>−</sub>c)2


16
⇔ a2 +b2+c2+abcX


cyc
1


b+c ≥


3(ab+bc+ca)


2 +



7(a<sub>−</sub>c)2


16


Theo bất đẳng thức Cauchy<sub>−</sub>Schwarz có:
X


cyc
1


b+c ≥


9
2(a+b+c)


B©y giê, ta cÇn chøng minh:


a2+b2+c2 + 9abc
2(a+b+c) ≥


3(ab+bc+ca)


2 +


7(a<sub>−</sub>c)2


16


Đặt a=c+x, b=c+y thì x<sub>≥</sub>y<sub>≥</sub>0, thay vào bất đẳng thức trên rồi biến đổi t-ơng


đ-ơng, ta có:


(11x2<sub>−</sub>32xy+ 32y2)c+ (x+y)(3x<sub>−</sub>4y)2<sub>≥</sub> 0


Bất đẳng thức trên hiển nhiên đúng. Từ đây ta suy ra điều phải chứng minh.
Vy, kmax = 7/16.


Bài toán 3.2.2. Cho a, b, c >0. Chøng minh r»ng:
a


b+c+
b
c+a +


c
a+b ≥


X


cyc


2ab


(c+a)(c+b)


H-ớng dẫn. Biến đổi t-ơng -ng ri s dng bt ng thc Schur.


Bài toán 3.2.3. Cho a, b, c >0. Chứng minh rằng:



a
b+c


2


+



b
c+a


2


+



c
a+b


2


3<sub>4</sub>




a2<sub>+</sub><sub>b</sub>2<sub>+</sub><sub>c</sub>2


ab+bc+ca



Bài toán 3.2.4. Cho a, b, c không âm. Chứng minh rằng:
a2


b2 <sub>+</sub><sub>c</sub>2 +


b2


c2<sub>+</sub><sub>a</sub>2 +


c2


a2<sub>+</sub><sub>b</sub>2 ≥


(a2<sub>+</sub><sub>b</sub>2<sub>+</sub><sub>c</sub>2<sub>)</sub>2


2(a2<sub>b</sub>2<sub>+</sub><sub>b</sub>2<sub>c</sub>2<sub>+</sub><sub>c</sub>2<sub>a</sub>2<sub>)</sub> ≥


(a+b+c)2


</div>
<span class='text_page_counter'>(16)</span><div class='page_container' data-page=16>

3.3 Nh×n theo h-ớng ng-ợc lại



Trong phn ny, chỳng ta s cựng nhỡn bất đẳng thức Nesbitt theo h-ớng ng-ợc lại,
hay nói cách khỏc l -a a


b+c +
b
c+a +


c



a+b vào thế yếu.


Bài toán 3.3.1. Cho a, b, c >0. Chøng minh r»ng:
a


b+c+
b
c+a +


c
a+b


a2<sub>+</sub><sub>bc</sub>


(b+c)2 +


b2<sub>+</sub><sub>ac</sub>


(c+a)2 +


c2 <sub>+</sub><sub>ab</sub>


(a+b)2


Lời giải. Ta có:


a2<sub>+</sub><sub>bc</sub>


(b+c)2



a
b+c =


(a<sub></sub>b)(a<sub></sub>c)
(b+c)2


Đặt x= <sub>(</sub><sub>b</sub><sub>+</sub>1<sub>c</sub><sub>)</sub>2, y =


1


(a+c)2, z =


1


(a+b)2, ta cÇn chøng minh:


x(a<sub>−</sub>b)(a<sub>−</sub>c) +y(b<sub>−</sub>a)(b<sub>−</sub>c) +z(c<sub>−</sub>a)(c<sub>−</sub>b)<sub>≥</sub>0


Giả sử a <sub>≥</sub>b <sub>≥</sub> c, ta dễ dàng suy ra x<sub>≥</sub>y <sub>≥</sub> z. Do đó, bất đẳng thức trên hin nhiờn
ỳng theo bt ng thc Schur suy rng.


Đẳng thức xảy ra khi và chỉ khi a=b =c.


Chỳ ý. Bt đẳng thức Schur suy rộng đ-ợc phát biểu nh- sau:


Định lý 3.1 (Bất đẳng thức Schur suy rộng). Với các số d-ơng a, b, c, x, y, z sao cho


(a, b, c) và (x, y, z)đều là các bộ đơn iu thỡ:


x(a<sub></sub>b)(a<sub></sub>c) +y(b<sub></sub>a)(b<sub></sub>c) +z(c<sub></sub>a)(c<sub></sub>b)<sub></sub>0



Bài toán 3.3.2. Cho a, b, c >0. Chứng minh rằng:
a


b+c+
b
c+a +


c
a+b


s


a3<sub>+</sub><sub>abc</sub>


(b+c)3 +


s


b3<sub>+</sub><sub>abc</sub>


(c+a)3 +


s


c3 <sub>+</sub><sub>abc</sub>


(a+b)3


Bài toán 3.3.3. Cho a, b, c >0. Chøng minh r»ng:


a


b+c+
b
c+a +


c
a+b


3
2




a2<sub>+</sub><sub>b</sub>2<sub>+</sub><sub>c</sub>2


ab+bc+ca


Bài toán 3.3.4. Chom, n >0; m < n; a, b, c >0; a, b, c<sub>∈</sub>(m, n). Chøng minh r»ng:
a


b+c+
b
c+a +


c
a+b ≤


3


2 +


(m<sub>−</sub>n)2


2m(m+n)


Bài toán 3.3.5. Cho a, b, c là độ dài ba cạnh của tam giác. Chứng minh rằng:
a


b+c+
b
c+a +


</div>
<span class='text_page_counter'>(17)</span><div class='page_container' data-page=17>

Lời giải. Không mất tính tổng quát, giả sử a <sub></sub>b <sub></sub>c >0 thìa+b <sub></sub>a+c<sub></sub>b+c.
Suy ra: c


a+b ≤
c
b+c,


b
c+a ≤


b
b+c,


a
b+c =


a


b+c.


Cộng vế các bất đẳng thức trên, ta có:
a


b+c +
b
c+a+


c
a+b ≤


a


b+c+ 1<1 + 1 = 2
Bất đẳng thức đã đ-ợc chứng minh xong.


Bài toán 3.3.6. Cho a, b, c là độ dài ba cạnh của tam giác. Chứng minh rằng:
a


b+c+
b
c+a +


c
a+b +


ab+bc+ca
a2<sub>+</sub><sub>b</sub>2<sub>+</sub><sub>c</sub>2 ≤



5
2


Lêi gi¶i. Ta cã:


2a
b+c +


2b
c+a +


2c


a+b −3 =


(a<sub>−</sub>b)2


(a+c)(b+c) +


(b<sub>−</sub>c)2


(b+a)(c+a)+


(c<sub>−</sub>a)2


(b+c)(b+a)
2<sub>−</sub> 2(ab+bc+ca)


a2<sub>+</sub><sub>b</sub>2<sub>+</sub><sub>c</sub>2 =



(a<sub>−</sub>b)2<sub>+ (</sub><sub>b</sub><sub>−</sub><sub>c</sub><sub>)</sub>2<sub>+ (</sub><sub>c</sub><sub>−</sub><sub>a</sub><sub>)</sub>2


a2<sub>+</sub><sub>b</sub>2<sub>+</sub><sub>c</sub>2


Ta phải chứng minh Sa(b<sub>−</sub>c)2<sub>+</sub><sub>S</sub><sub>b(</sub><sub>c</sub><sub>−</sub><sub>a</sub><sub>)</sub>2<sub>+</sub><sub>S</sub><sub>c(</sub><sub>a</sub><sub>−</sub><sub>b</sub><sub>)</sub>2 <sub>≥</sub><sub>0</sub><sub>. Trong đó:</sub>


Sa= 1−


a2<sub>+</sub><sub>b</sub>2<sub>+</sub><sub>c</sub>2


(a+b)(a+c), Sb = 1−


a2<sub>+</sub><sub>b</sub>2<sub>+</sub><sub>c</sub>2


(b+a)(b+c), Sc = 1


a2<sub>+</sub><sub>b</sub>2<sub>+</sub><sub>c</sub>2


(c+a)(c+b)


Không mất tính tổng quát, giả sử a<sub></sub> b<sub></sub>cthì Sa0.


Vỡ a, b, c là độ dài ba cạnh của tam giác nên:
Sb =


a(b+c<sub>−</sub>a) +c(b<sub>−</sub>c)
(b+a)(b+c) ≥


c(b<sub>−</sub>c)
(b+a)(b+c)



Sc =


a(b+c<sub>−</sub>a) +b(c<sub>−</sub>b)
(c+a)(c+b) ≥


b(c<sub>−</sub>b)
(c+a)(c+b)


a<sub>−</sub>c
a<sub>−</sub>b ≥


b
c ≥


a+b
a+c
Từ các bất đẳng thức trên suy ra:


Sa(b<sub>−</sub>c)2+Sb(c−a)2+Sc(a−b)2 ≥Sb(c−a)2+Sc(a−b)2
≥ (a<sub>−</sub>b)2



b2


c2Sb +Sc




≥ (ab)



2


c2




b2<sub>c</sub><sub>(</sub><sub>b</sub><sub></sub><sub>c</sub><sub>)</sub>


(b+a)(b+c) +


c2<sub>b</sub><sub>(</sub><sub>c</sub><sub></sub><sub>b</sub><sub>)</sub>


(c+a)(c+b)




= (ab)


2<sub>(</sub><sub>b</sub><sub></sub><sub>c</sub><sub>)</sub><sub>b</sub>


(b+a)(b+c)



b
c


a+b
a+c





0.
Đẳng thức xảy ra khi và chỉ khi a=b =c.


</div>
<span class='text_page_counter'>(18)</span><div class='page_container' data-page=18>

3.4 So s¸nh

a
b+c

+



b
c+a

+



c
a+b



b+c
a

+



c+a
b

+



a+b
c


Nh- chúng ta đã biết a
b+c +


b
c+a+


c


a+b ≥


3
2;


b+c
a +


c+a
b +


a+b


c ≥6. Vì hai bất đẳng thức


trên cùng chiều nên ta không dễ dàng đ-a ra ngay đ-ợc phép so sánh giữa chúng. Sau
đây, tôi xin đ-a ra một vi vớ d so sỏnh.


Bài toán 3.4.1. Cho a, b, c >0. Chøng minh r»ng:
b+c


a +


c+a


b +


a+b
c ≥4




a
b+c+


b
c+a +


c
a+b




Lời giải. Theo bất đẳng thức Cauchy<sub>−</sub>Schwarz có:
a


b +
a
c ≥


4a
b+c;


b
a +


b
c ≥


4b
c+a;



c
a +


c
b ≥


4c
a+b.
Cộng vế các bất đẳng thức trên ta cú iu phi chng minh.


Đẳng thức xảy ra khi và chỉ khi a=b =c.


Bài toán 3.4.2. Cho a, b, c >0. Chøng minh r»ng:
b+c


a +


c+a


b +


a+b


c ≥


a
b+c +


b


c+a +


c
a+b +


9
2


Bµi to¸n 3.4.3 (ViƯt Nam TST 2006). Cho a, b, c<sub>∈</sub>[1,2]. Chứng minh rằng:


(a+b+c)



1
a +
1
b +
1
c

6

a
b+c+


b
c+a +


c
a+b





Bài toán 3.4.4. Cho a, b, c >0. Chøng minh r»ng:
r


b+c


a +


r
c+a


b +


r
a+b


c ≥2
r


a
b+c+


r
b
c+a +


r
c


a+b


!


Bình luận. Sau một số ví dụ đ-ợc đ-a ra để so sánh a
b+c+


b
c+a+


c
a+b vµ


b+c
a +


c+a
b +


a+b
c ,


ta thÊy a
b+c +


b
c+a +


c



a+b có phần bị yếu thế hơn.


3.5 Nhng bt ng thc lng ghộp



</div>
<span class='text_page_counter'>(19)</span><div class='page_container' data-page=19>

Bài toán 3.5.1. Cho a, b, c không âm vàn > 0. Chứng minh rằng:
a


b+c+
b
c+a +


c
a+b +


n2<sub>(</sub><sub>ab</sub><sub>+</sub><sub>bc</sub><sub>+</sub><sub>ca</sub><sub>)</sub>


a2<sub>+</sub><sub>b</sub>2<sub>+</sub><sub>c</sub>2 ≥2n


Lời giải. Khơng mất tính tổng qt, giả sử a+b+c= 1; và đặt ab+bc+ca=q. Bất
đẳng thức t-ơng đ-ơng với:


3abc+ 1<sub>−</sub>2q
q<sub>−</sub>abc +


n2<sub>q</sub>


1<sub>−</sub>2q ≥2n
Ta cã:


V T <sub>≥</sub> 1−2q



q +


n2<sub>q</sub>


1<sub>−</sub>2q ≥2n
Bất ng thc ó -c chng minh xong.


Bài toán 3.5.2. Cho a, b, c >0. Chøng minh r»ng:


a
b+αc+


b
c+βa +


c
a+γb


1
3 +


αca+βab+γbc


(a+b+c)2




≥1



Lời giải. Theo bất đẳng thức Cauchy<sub>−</sub>Schwarz có:


(a+b+c)2


=


r
a
b+αc


p


a(b+αc) +


s
b
c+βa


p


b(c+βa) +


r
c
a+γb


p


c(a+γb)



!2





a
b+αc +


b
c+βa +


c
a+γb




(ab+bc+ca+αca+βab+γbc)


Suy ra,


1<sub>≤</sub>



a
b+αc +


b
c+βa +



c
a+γb


1
3 +


αca+βab+γbc


(a+b+c)2




Bất đẳng thức đã đ-ợc chứng minh xong.


Bài tốn 3.5.3. Choa, b, c khơng âm. Tìm hằng số k d-ơng lớn nhất để bất đẳng thức
sau đúng:


a
b+c+


b
c+a +


c
a+b +k


(a+b)(b+c)(c+a)


a2<sub>+</sub><sub>b</sub>2<sub>+</sub><sub>c</sub>2 ≥k+



3
2


Bài tốn 3.5.4. Cho a, b, ckhơng âm. Tìm điều kiện cho các số d-ơng k, l để bất đẳng
thức sau đúng:


a
b+c+


b
c+a +


c
a+b +k


(a+b)(b+c)(c+a)


a2<sub>+</sub><sub>b</sub>2<sub>+</sub><sub>c</sub>2 +l


a2<sub>+</sub><sub>b</sub>2<sub>+</sub><sub>c</sub>2


(a+b+c)2 ≥


3


2 +k+


l


</div>
<span class='text_page_counter'>(20)</span><div class='page_container' data-page=20>

Bài toán 3.5.5. Cho a, b, c không âm. Chøng minh r»ng:


a


b+c+
b
c+a +


c
a+b +


4(a+b)(b+c)(c+a)


a3<sub>+</sub><sub>b</sub>3<sub>+</sub><sub>c</sub>3 ≥5


Lời giải. Khơng mất tính tổng quát, giả sử a+b+c = 1; và đặt ab+bc+ca = q,
abc=r. Bất đẳng thức t-ơng đ-ơng với:


1<sub>−</sub>2q+ 3r
q<sub>−</sub>r +


4(q<sub>−</sub>r)
1<sub>−</sub>3q<sub>−</sub>3r ≥5
Theo bất đẳng thức AM <sub>−</sub>GM có:


1<sub>−</sub>2q+ 3r
q<sub>−</sub>r +


4(q<sub>−</sub>r)
1<sub>−</sub>3q<sub>−</sub>3r =


q


q<sub>−</sub>r +


1<sub>−</sub>3q+ 3r
q<sub>−</sub>r +


4(q<sub>−</sub>r)
1<sub>−</sub>3q<sub>−</sub>3r ≥5
Bất đẳng thức đã đ-ợc chứng minh xong.


Ta có bài toán tổng quát.


Bài toán 3.5.6. Cho a, b, c không âm vàn > 0. Chứng minh rằng:
a


b+c+
b
c+a +


c
a+b +


n2<sub>(</sub><sub>a</sub><sub>+</sub><sub>b</sub><sub>)(</sub><sub>b</sub><sub>+</sub><sub>c</sub><sub>)(</sub><sub>c</sub><sub>+</sub><sub>a</sub><sub>)</sub>


a3<sub>+</sub><sub>b</sub>3<sub>+</sub><sub>c</sub>3 ≥1 + 2n


Tõ nhËn xÐt (a+b+c)(ab+bc+ca)<sub>≥</sub>(a+b)(b+c)(c+a) +abc, ta có bài toán sau:


Bài toán 3.5.7. Cho a, b, c không âm. Chứng minh rằng:
a



b+c+
b
c+a +


c
a+b +


n2<sub>(</sub><sub>a</sub><sub>+</sub><sub>b</sub><sub>+</sub><sub>c</sub><sub>)(</sub><sub>ab</sub><sub>+</sub><sub>bc</sub><sub>+</sub><sub>ca</sub><sub>)</sub>


a3<sub>+</sub><sub>b</sub>3<sub>+</sub><sub>c</sub>3 1 + 2n


Bài to¸n 3.5.8. Cho a, b, c >0. Chøng minh r»ng:
a


b+c+
b
c+a +


c
a+b +


abc


2(a3<sub>+</sub><sub>b</sub>3<sub>+</sub><sub>c</sub>3<sub>)</sub>


5
3


Bài toán 3.5.9. Cho a, b, c >0. Chứng minh rằng:
a



b+c+
b
c+a +


c
a+b +


4abc


(a+b)(b+c)(c+a) 2


Bài toán 3.5.10. Cho a, b, c không âm. Chứng minh rằng:


a
b+c


3


+



a
b+c


3


+




a
b+c


3


+ 5abc


(a+b)(b+c)(c+a)


a2<sub>+</sub><sub>b</sub>2<sub>+</sub><sub>c</sub>2


</div>
<span class='text_page_counter'>(21)</span><div class='page_container' data-page=21>

Ta xét một số bài toán có căn thức.


Bài to¸n 3.5.11. Cho a, b, c >0. Chøng minh r»ng:
r


a
b+c +


r
b
c+a +


r
c
a+b + 3


r



3(ab+bc+ca)


a2<sub>+</sub><sub>b</sub>2<sub>+</sub><sub>c</sub>2


72
2


Lời giải. Không mất tính tổng quát, giả sử a <sub></sub>b <sub></sub>c, ta sẽ chứng minh đ-ợc:
r


b
c+a +


r
c
a+b


r
b+c


a


Lại có: ab+bc+ca<sub></sub>a(b+c) và a2<sub>+</sub><sub>b</sub>2<sub>+</sub><sub>c</sub>2 <sub></sub><sub>a</sub>2<sub>+ (</sub><sub>b</sub><sub>+</sub><sub>c</sub><sub>)</sub>2 <sub>nên:</sub>


V T <sub>≥</sub>
r


a
b+c+



r
b+c


a + 3
s


3a(b+c)


a2<sub>+ (</sub><sub>b</sub><sub>+</sub><sub>c</sub><sub>)</sub>2 =x+


3√3


x2<sub>−</sub><sub>2</sub>


Trong đó, x=p


a/(b+c) +p


(b+c)/a<sub>≥</sub>2.
B©y giê, ta cÇn chøng minh:


x+ 3


3


x2<sub>−</sub><sub>2</sub> ≥



7√2
2


Nếu x<sub>≥</sub> 7√<sub>2</sub>2, bất đẳng thức hiển nhiên đúng. Nếux<sub>≤</sub> 7√<sub>2</sub>2, ta có:


27


x2<sub>−</sub><sub>2</sub> −


7√2


2 −x


!2


= (x−2


2)2<sub>(19 + 6</sub>√<sub>2</sub><sub>x</sub><sub>−</sub><sub>2</sub><sub>x</sub>2<sub>)</sub>


2(x2<sub>−</sub><sub>2)</sub> ≥0(do x≤


7√2
2 )


Bất đẳng thức đã đ-ợc chng minh xong.


Đẳng thức xảy ra khi và chỉ khi (a, b, c)<sub></sub>(3 + 22,1,0).
Ta có bài toán tổng quát hơn.



Bài toán 3.5.12. Cho a, b, c >0. Chứng minh rằng:
r


a
b+c +


r
b
c+a +


r
c


a+b +k.
r


(ab+bc+ca)


a2<sub>+</sub><sub>b</sub>2<sub>+</sub><sub>c</sub>2 min<sub>x</sub><sub></sub><sub>2</sub>




x+<sub></sub> k


x2<sub></sub><sub>2</sub>




Bài toán 3.5.13. Cho a, b, c >0. Chøng minh r»ng:
a



b+c+
b
c+a +


c
a+b + 2




abc


(a+b)(b+c)(c+a)


2/3


≥ 2


Bµi toán 3.5.14. Cho a, b, c<sub></sub>0và k <sub></sub>4. Chứng minh r»ng:
r


a
b+c +


r
b
c+a +


r
c


a+b +k


s


(a+b)(b+c)(c+a)
(a+b+c)3 ≥2




</div>
<span class='text_page_counter'>(22)</span><div class='page_container' data-page=22>

4 Bµi tËp áp dụng



Để kết thúc bài viết, tôi xin nêu ra một số bài tập t-ơng tự và áp dụng.


Bài tập 4.1. Cho a, b, c, d >0 tho¶ m·nabcd= 1. Chøng minh r»ng:


1


a(1 +b)+
1


b(1 +c)+
1


c(1 +d)+
1


d(1 +a) ≥2


Bµi tËp 4.2. Cho a, b, c >0. Chøng minh r»ng:
a3<sub>b</sub>



1 +ab2 +


b3<sub>c</sub>


1 +bc2 +


c3<sub>a</sub>


1 +ca2 ≥


abc(a+b+c)
1 +abc


Bµi tËp 4.3 (IMO 1995). Choa, b, c >0 thoả mÃnabc= 1. Chứng minh rằng:


1


a3<sub>(</sub><sub>b</sub><sub>+</sub><sub>c</sub><sub>)</sub>+


1


b3<sub>(</sub><sub>c</sub><sub>+</sub><sub>a</sub><sub>)</sub>+


1


c3<sub>(</sub><sub>a</sub><sub>+</sub><sub>b</sub><sub>)</sub>


3
2



Bài tËp 4.4. Cho a, b, c >0. Chøng minh r»ng:


1


b(b+a) +
1


c(c+b)+
1


a(a+c) ≥


9


2(ab+bc+ca)


Bµi tËp 4.5. Cho a, b, c >0. Chøng minh r»ng:
r


a
b+c+


r
a
b+c+


r
a
b+c ≥2



s


abc


(a+b)(b+c)(c+a)+ 1


Bµi tËp 4.6. Cho a, b, c >0 tho¶ m·na2<sub>+</sub><sub>b</sub>2<sub>+</sub><sub>c</sub>2 <sub>= 3</sub><sub>. Chøng minh r»ng:</sub>


a3<sub>+</sub><sub>abc</sub>


(b+c)2 +


b3<sub>+</sub><sub>abc</sub>


(c+a)2 +


c3<sub>+</sub><sub>abc</sub>


(a+b)2 ≥


3
2


Bµi tËp 4.7. Cho a, b, c >0. Chøng minh r»ng:
a


(bx+cy)3 +


b



(cx+ay)3 +


c


(ax+by)3 ≥


9


(x+y)3<sub>(</sub><sub>ab</sub><sub>+</sub><sub>bc</sub><sub>+</sub><sub>ca</sub><sub>)</sub>


Bµi tËp 4.8 (JBMO, 2003). Cho a, b, c ><sub>−</sub>1. Chøng minh r»ng:


1 +x2


1 +y+z2 +


1 +y2


1 +z+x2 +


1 +z2


1 +x+y2 ≥2


Bµi tËp 4.9. Cho a, b, c, k không âm. Chứng minh rằng:


1 + ka



b+c 1 +
kb


c+d 1 +
kc


d+a 1 +
kd
a+b




</div>
<span class='text_page_counter'>(23)</span><div class='page_container' data-page=23>

[1] Phạm Kim Hùng, Sáng tạo bất đẳng thức, NXB Tri Thức, 2006.


[2] Phạm Văn Thuận, Lê Vĩ,Bất đẳng thức Suy luận và Khám phá, NXB ĐHQGHN,
2007.


[3] Nguyễn Vũ L-ơng, Nguyễn Ngọc Thắng, Các bài giảng về bất đẳng thức
Bunhiacopxki, NXB ĐHQGHN, 2007.


[4] Titu Andresscu, Vasile Cirtoaje, Gabriel Dospinescu, Mircea Lascu, Old and New
Inequality.


</div>

<!--links-->

Tài liệu bạn tìm kiếm đã sẵn sàng tải về

Tải bản đầy đủ ngay
×